Đến nội dung

Tru09 nội dung

Có 629 mục bởi Tru09 (Tìm giới hạn từ 28-04-2020)



Sắp theo                Sắp xếp  

#414128 $\frac{1}{a+b}+\frac{1}{b+c...

Đã gửi bởi Tru09 on 21-04-2013 - 16:41 trong Bất đẳng thức và cực trị

Bài làm :
$A =3 +\sum \frac{c}{a+b} +3(ab+bc+ca) \geq 3 +\frac{1}{2(ab+bc+ca)} +3(ab+bc+ca) =3+\frac{1}{6(ab+bc+ca)} +\frac{1}{3(ab+bc+ca)} +3(ab+bc+ca) \geq 3 +\frac{1}{2} +2 =\frac{11}{2}$

dấu = khi $a=b=c=\frac{1}{3}$




#413902 Chứng minh rằng $H$ thuộc trung trực AB

Đã gửi bởi Tru09 on 20-04-2013 - 18:24 trong Hình học

Bài toán :

Cho $\Delta ABC . AD ,BF$ là các đường đối trung .Trung tuyến $AJ , BK$  của  $\Delta ABF$ và $\Delta ABD$ cắt nhau tại $H$ .Chứng minh rằng $H$ thuộc trung trực $AB$




#413879 [MSS2013] - Trận cuối - Hình học

Đã gửi bởi Tru09 on 20-04-2013 - 16:41 trong Thi giải toán Marathon cấp THCS 2013

Bài làm :
Lấy X là trung điểm BC

Ta có $OX \perp BC$

Xét tứ giác $OXDI$ có $\angle OXI =\angle ODI =90^o$

$\Rightarrow OXDI$ là tứ giác nội tiếp

$\Rightarrow \angle ODI =\angle IXD $

$\Rightarrow \angle AOM =\angle XCD (1)$

Xét $\Delta AMO$  và $\Delta CDX$ có :

(1) và $\angle BAD =\angle BCD$

$\Rightarrow \Delta AMO$ ~ $\Delta BCD$

$\Rightarrow \frac{MO}{XD} =\frac{AO}{\frac{BC}{2}} (2)$

Xét $\Delta AON$ và  $\Delta BXD$

có :$\angle AON =\angle BXD$

$\angle OAN =\angle BCD$

$\Rightarrow \Delta AON $~ $\Delta BXD$

$\Rightarrow \frac{ON}{XD} =\frac{AO}{\frac{BC}{2}} (3)$

Từ (2) và (3) 

$\Rightarrow ON =OM$

MSS cuối.PNG

Các chỗ màu đỏ sai

Điểm bài: 6

S = 16+3*6 = 34




#412575 ĐỀ THI THỬ VÀO 10 CHUYÊN KHTN ĐỢT 2 MÔN TOÁN CHUYÊN

Đã gửi bởi Tru09 on 14-04-2013 - 15:47 trong Tài liệu - Đề thi

TRƯỜNG ĐẠI HỌC KHOA HỌC TỰ NHIÊN         ĐỀ THI KIỂM TRA KIẾN THỨC LỚP 9 (đợt 2)

                                                                                                    

 TRƯỜNG THPT CHUYÊN KHTN                                          NĂM HỌC 2012-2013

                                             

                                               Môn: TOÁN HỌC (vòng 2)

                                               Thời gian làm bài: 150 phút

                                               Ngày thi: 14/04/2013

 

Câu $1$: ($3$ điểm)

$a)$ Giải phương trình: $x^{3}-(x^{2}-x)\sqrt{x+2}-x-2=0$

$b)$ Giải hệ phương trình: $\left\{\begin{matrix} x+y+z+t=0 & & & \\ x^{2}+y^{2}+z^{2}+t^{2}=4 & & & \\ x^{3}+y^{3}+z^{3}+t^{3}=0 & & & \\ x^{4}+y^{4}+z^{4}+t^{4}=4 & & & \end{matrix}\right.$

Câu $2$: ($3$ điểm)

$a)$ Cho $a,b,c$ là các số thực dương thỏa mãn $a+2b+3c=14$. Tìm giá trị lớn nhất của: $A=\sqrt{a+3}+2\sqrt{b+2}+3\sqrt{c+1}$

$b)$ Tìm số nguyên tố $p$ sao cho $2p^{2}-3$ và $2p^{2}+3$ là các số nguyên tố.

Câu $3$: ($3$ điểm)

Cho tam giác nhọn $ABC$ có $AB<AC$. Các đường cao $AE,BF,CK$ của tam giác $ABC$ cắt nhau tại $H$. Gọi $O$ là tâm đường tròn ngoại tiếp tam giác $ABC$. Đường thẳng đi qua $A$ và $O$ cắt $KF$ tại $M$, cắt đường tròn ngoại tiếp tam giác $ABC$ tại $N$ ($N$ khác $A$)

$a)$ Chứng minh tứ giác $MNEH$ là tứ giác nội tiếp

$b)$ Gọi $P$ là giao của $KE$ và $BF$. Chứng minh rằng $HP.BF=HF.BP$

Câu $4$: ($1$ điểm)

Có ba chiếc hộp, hộp thứ nhất có $17$ viên bi, hộp thứ hai có $22$ viên, hộp thứ ba có $24$ viên. Chọn hai hộp bất kỳ và lấy ra từ mỗi hộp ra $1$ viên bi rồi cho vào hộp còn lại (nếu một trong hai chiếc hộp chọn ra không có viên bi nào bỏ hộp ra và chọn hộp còn lại). Hỏi bằng cách đó có thể chuyển tất cả các viên bi vào một hộp hay không?

Rảnh ngồi post :3

Bài 1 :

$PT \Leftrightarrow (x-\sqrt{x+2})(x^2 +\sqrt{x+2}) =0$

$\Leftrightarrow x =\sqrt{x+2} \Leftrightarrow x^2 -x-2 =0 \Leftrightarrow x =2 $

b, dùng cauchy :
$x^4 +y^4 +z^4 +t^4 +4 \geq 2(x^2+y^2+z^2+y^2) =8$

$\Leftrightarrow x^4+y^4 +z^4 +t^4 \geq 4$

$\Rightarrow dấu = xảy ra \Leftrightarrow x=\pm 1 , y =\pm 1 , z =\pm1 . t =\pm 1$

Mà $x+y+z+y =0$

$\Rightarrow$  trong 4 số $x,y,z,t $có 2 số $=-1$  2 số $=1$

Bài 2 

a$A=\sqrt{a+3} +2\sqrt{b+2} +3\sqrt{c+1} \leq \sqrt{(1+2+3)(a+3+2b+4+3c+3)} =12$

Dấu = khi $a=1 ,b=2 ,c=3$

b Bình phương của 1 số chỉ $\equiv \pm 1 mod 5$

$\Rightarrow$ khi $p^2 \equiv 1 mod 5$ thì $2p^2 +3 \vdots 5 \Leftrightarrow 2p^2 +3 =5 \Leftrightarrow p =1$ loại

Khi $p^2 \equiv -1 mod 5 \Rightarrow 2p^2 -3 \vdots 5 \Leftrightarrow p =2 :\text{Chuẩn}$

Vậy p =2

Bài 3 :

a, Dễ có $AM.,AN = AK.AB  và AH .AE =AK .AB$

$\Rightarrow$ dpcm

b,Dễ thấy KH là phân giác $\angle EKF$

và KB là phân giác ngoài $\angle EKF$

$\Rightarrow$ DPCM

Hình gửi kèm

  • Hình.PNG



#412113 [MSS2013] - Trận 26 - PT, HPT

Đã gửi bởi Tru09 on 12-04-2013 - 21:08 trong Thi giải toán Marathon cấp THCS 2013

 

Đề bài: Giải hệ phương trình sau:

$$\begin{cases} x^4+y^4+4x^2y^2-3xy(x^2+y^2)=0 \\ x+x^3+x^2y-xy^2-y^3=1 \end{cases}$$

 

Đề của 

Nguyen Duc Thuan

Bài làm  :
Từ phương trình đầu 

$x^4+y^4+4x^2y^2-3xy(x^2+y^2)=0$

$\Leftrightarrow x^4+y^4 +6x^2y^2 -4xy(x^2+y^2) +xy(x^2+y^2)-2x^2y^2 =0$

$\Leftrightarrow (x-y)^4 +xy[(x-y)^2+2xy) -2x^2y^2 =0$

$\Leftrightarrow (x-y)^4 +xy(x-y)^2 =0$

$\Leftrightarrow (x-y)^2[(x-y)^2 +xy)] =0$

$\Leftrightarrow x=y$ hoặc $x^2 -xy+y^2 =0$

Mà $x^2 +y^2 -xy =(x-\frac{1}{2}y)^2 +\frac{3y^2}{4} > 0 \forall x,y$

$\Rightarrow x=y$

Thay vào pt 2  $x +x^3 +x^2y -xy^2 -y^3 =1$

$\Rightarrow x=1 \Rightarrow x=y=1$

 

Chỗ màu đỏ chưa chính xác

Điểm bài: 9

S = 26+9*3 = 53




#411249 [MSS2013] - Trận 25 - BĐT

Đã gửi bởi Tru09 on 08-04-2013 - 12:48 trong Thi giải toán Marathon cấp THCS 2013

Chỗ này là sao ta, bất đẳng thức này khi đẳng thức xảy ra đâu phải tại $a=b=c$ ?

Đây là 1 bổ đề quen thuộc, nhưng hình như cách chứng minh của bạn bị sai rồi :D

 

2 cái bạn tô đậm là 2 bổ đề khác nhau bạn à :| . cái  trên là bunia bạn nhé , còn cái dưới thì biến đổi tương đương :3




#411151 Đề thi HSG toán 9 tỉnh Bắc Ninh năm 2012-2013

Đã gửi bởi Tru09 on 07-04-2013 - 21:06 trong Tài liệu - Đề thi

      UBND TỈNH BẮC NINH                       ĐỀ THI CHỌN HỌC SINH GIỎI CẤP TỈNH

SỞ GIÁO DỤC VÀ ĐÀO TẠO                                    NĂM HỌC 2012 – 2013

                                                                                    MÔN THI: TOÁN – LỚP 9 –THCS

 

ĐỀ CHÍNH THỨC

                                                                 Thời gian làm bài: 150 phút (không kể thời gian giao đề)

 

                                                                                  Ngày thi 29 tháng 3 năm 2013

                                                                                        = = = = = = = = = = = =

 

Câu 1. (4,0 điểm)

          Cho biểu thức: $P=\frac{a^2 -\sqrt{a}}{a+\sqrt{a} +1} -\frac{3a-2\sqrt{a}}{\sqrt{a}} +\frac{a-4}{\sqrt{a} -2}$

1.     Rút gọn biểu thức P

2.     Tìm giá trị nhỏ nhất của biểu thức P

Câu 2. (4,0 điểm)                                                      

          1.  Trong mặt phẳng tọa độ (Oxy), cho parabol (P) có phương trình y = x2 và đường thẳng d có phương trình y = kx+1 (k là tham số). Tìm k để đường thẳng d cắt parabol (P) tại hai điểm phân biệt M, N sao cho MN=$2\sqrt{10}.$

          2.  Giải hệ phương trình: $\begin{Bmatrix} (x+y)(x+z) =12 & \\ (y+x)(y+z) =15 & \\ (z+y)(z+x) =20 & \end{Bmatrix}$

 (Với x, y, z là các số thực dương).

Câu 3. (3,0 điểm)

1.     Giải phương trình nghiệm nguyên: $x^4 -2y^4 -x^2y^2 -4x^2 -7y^2  -5 =0.$

2.     Cho ba số a, b, c thỏa mãn $a+b+c =1; a^2+b^2+c^2 =1; a^3+b^3 +c^3 =1$

Chứng minh rằng: $a^{2013} +b^{2013} +c^{2013}.$

Câu 4. (6,0 điểm)

          Cho đường tròn (O; R), đường thẳng d không đi qua O cắt đường tròn tại hai điểm A, B. Từ một điểm M tùy ý trên đường thẳng d và nằm ngoài đường tròn (O), vẽ hai tiếp tuyến MN, MP của đường tròn (O) (N, P là hai tiếp điểm).

1.     Dựng điểm M trên đường thẳng d sao cho tứ giác MNOP là hình vuông.

          2.   Chứng minh rằng tâm của đường tròn đi qua ba điểm M, N, P luôn thuộc đường thẳng cố định khi M di động trên đường thẳng d.

Câu 5. (3,0 điểm)

          1. Tìm hai số nguyên dương ab thỏa mãn  $a^2 +b^2$ =[a,b] +7(a,b)

          (với [a,b] = BCNN(a,b), (a,b) = ƯCLN(a,b)).

          2. Cho tam giác ABC thay đổi có AB = 6, AC = 2BC. Tìm giá trị lớn nhất của diện tích tam giác ABC.

------------------------Hết--------------------------




#411092 $Sin\frac{A}{2}<\frac{a}...

Đã gửi bởi Tru09 on 07-04-2013 - 17:56 trong Công thức lượng giác, hàm số lượng giác

Chứng minh rằng:

1.$Sin\frac{A}{2}<\frac{a}{2\sqrt{bc}}$

2.$Sin\frac{A}{2}Sin\frac{B}{2}Sin\frac{C}{2}\leqslant \frac{1}{8}$

Với A,B,C là ba góc và a,b,c là ba cạnh đối diện A,B,C

 

1Vẽ Tam giác ABC và đường phân giác AD

Từ B kẻ đường thẳng vuôn góc AD tại I

Ta có $Sin\frac{A}{2} =\frac{BI}{AB} < \frac{BD}{AB} =\frac{a}{b+c} <\frac{a}{2\sqrt{bc}}$

2 /thì nhân hết lại  $\Rightarrow$ DPCM




#410885 Cho tam giác ABC Có I là tâm đường tròn ngoại tiếp,AD là đường phân giác tron...

Đã gửi bởi Tru09 on 06-04-2013 - 21:26 trong Hình học phẳng

Dễ thấy khi kẻ $AH \perp BC $

$\Rightarrow \angle BAH =\angle IAC$ cùng phụ với $\angle ABC$

Mà từ giả thiết  $\Rightarrow \angle DAI =\angle DAI'$ và $\angle DAB =\angle  DAC $

$\Rightarrow  \angle I'AB =\angle IAC$

Từ 2 điều trên  $\Rightarrow AI'D$ thẳng hàng $\Rightarrow AI' \perp BC$




#410645 [MSS2013] - Trận 25 - BĐT

Đã gửi bởi Tru09 on 05-04-2013 - 21:32 trong Thi giải toán Marathon cấp THCS 2013

Đề của duaconcuachua98

Cho $a,b,c>0$. Chứng minh rằng

$$S=\frac{a}{b}+\frac{b}{c}+\frac{c}{a}+\frac{a+b+c}{\sqrt{a^{2}+b^{2}+c^{2}}}\geq 3+\sqrt{3}$$

 

Bài làm

Bổ đề   phụ :

---$a^3 +b^3 +c^3 \geq 3abc$

$\Leftrightarrow (a+b+c)((a-b)^2 +(b-c)^2 +(c-a)^2).\frac{1}{2} \geq 0$ với mọi $a,b,c >0$

--- $\frac{a^2}{x} +\frac{b^2}{y} +\frac{c^2}{z} \geq \frac{(a+b+c)^2}{x+y+z}$

Ta có $(x+y+z )(\frac{a^2}{x} +\frac{b^2}{y} +\frac{c^2}{z}) \geq (a+b+c)^2 \quad (*)$

$\Rightarrow  \frac{a^2}{x} +\frac{b^2}{y} +\frac{c^2}{z} \geq \frac{(a+b+c)^2}{x+y+z}$

----$a^2+b^2+c^2 \geq ab+bc+ca$

$\Leftrightarrow (a-b)^2 +(b-c)^2 +(c-a)^2 \geq 0$

Luôn đúng với mọi $a,b,c $

Quay trở lại bài toán :

Ta có :
$\frac{a}{b} +\frac{b}{c} +\frac{c}{a} =\frac{a^2}{ab} +\frac{b^2}{bc} +\frac{c^2}{ca} \geq \frac{(a+b+c)^2}{ab+bc+ca} =2 +\frac{a^2+b^2+c^2}{ab+bc+ca}$

Vậy $S \geq 2 +\frac{a^2+b^2+c^2}{ab+bc+ca} (1-\frac{\sqrt{3}}{2}) +\frac{\sqrt{3}}{2} .\frac{a^2+b^2+c^2}{ab+bc+ca} +\frac{a+b+c}{2(\sqrt{a^2+b^2+c^2})} +\frac{a+b+c}{2(\sqrt{a^2+b^2+c^2})} $

Sử dụng bđt AMGM 

$\Rightarrow  \frac{\sqrt{3}}{2} .\frac{a^2+b^2+c^2}{ab+bc+ca} +\frac{a+b+c}{2(\sqrt{a^2+b^2+c^2})} +\frac{a+b+c}{2(\sqrt{a^2+b^2+c^2})} \geq 3\sqrt[3]{\frac{\sqrt{3}(a+b+c)^2}{8(ab+bc+ca)}} \geq 3\sqrt[3]{\frac{\sqrt{3}.3(ab+bc+ca)}{8(ab+bc+ca)}} =3\sqrt[3]{\frac{\sqrt{3^3}}{2^3}} =3.\frac{\sqrt{3}}{2}$

Vậy $S \geq 2 + 1 -\frac{\sqrt{3}}{2} +3 \frac{\sqrt{3}}{2} =3 +\sqrt{3}$

Dấu = xảy ra $\Leftrightarrow a=b=c$

 

Nhận xét:

Phần Bổ đề và bài làm nên để tách riêng một chút để dễ nhận biết.

Bước $(*)$ ,BĐT Cauchy-Schwarz bộ 3 số không được sử dụng ở bậc THCS.

Nhìn chung lời giải này hay và khá đặc sắc.

 

Điểm: 26 + 9*3 = 53




#410231 $\left\{\begin{matrix}x^2-y+1=0 &...

Đã gửi bởi Tru09 on 03-04-2013 - 22:00 trong Phương trình - hệ phương trình - bất phương trình

Trong sách toán nâng cao phát triển tập 2 nhá , hoặc bạn lên google cũng xong :)




#410209 Chứng minh: $BH=AC$

Đã gửi bởi Tru09 on 03-04-2013 - 21:07 trong Hình học

Định lý ceva ở đây  bạn thay vào là ra cái hệ thức của mình thôi :)




#410127 Chứng minh: $BH=AC$

Đã gửi bởi Tru09 on 03-04-2013 - 16:27 trong Hình học

Dễ đàng suy ra từ định  lý ceva 

$\Rightarrow \frac{BH}{CH} =\frac{DB}{DA} =\frac{BC}{AC}$

$\Rightarrow BG =\frac{CH.BC}{AC} =AC$




#410084 $\frac{2\sqrt{3}}{\sqrt{x+1...

Đã gửi bởi Tru09 on 03-04-2013 - 11:18 trong Phương trình, hệ phương trình và bất phương trình

Giải như sau :
$PT \Leftrightarrow 2\sqrt{3} =\sqrt{x+1}(\sqrt{x+3}-\sqrt{x})$

$\Leftrightarrow 2\sqrt{3} =\frac{3\sqrt{x+1}}{\sqrt{x+3} +\sqrt{x}}$

$\Leftrightarrow 2(\sqrt{x+3} +\sqrt{x}) =\sqrt{3x+3}$

$\Leftrightarrow  2\sqrt{x} +\frac{4x +12 -3x-3}{2\sqrt{x+3} +\sqrt{3x+3}} =0$

$\Leftrightarrow 2\sqrt{x} +\frac{x+9}{2\sqrt{x+3} +\sqrt{3x+3}} =0$

Vô lý  $\Rightarrow$ pt vô nghiệm




#410078 Cho phương trình $x^{2}-97x+a=0$ có các nghiệm là lũy thừ...

Đã gửi bởi Tru09 on 03-04-2013 - 11:05 trong Phương trình, hệ phương trình và bất phương trình

Giải như sau :

Nếu các nghiệm của pt 1 là lũy thừa bậc 4 của pt 2 thì

Gọi nghiệm của pt 1 là $x^4 , y^4$ với x,y là nghiệm của pt 2

Ta có $x^4 +y^4 =97$

$x^4 .y^4 =a =b^4$

và $x+y =1$

$\Rightarrow 97=(x+y)^4 -4xy ((x+y)^2-2xy)-6x^2y^2 =1-4b(1-2b)-6b^2$

$\Rightarrow 97 =1-4b +2b^2$

$\Rightarrow b^2 -2b -48 =0$

$\Rightarrow b =8$ hoặc $b =-6$

Thay vào pt 1 và 2  rồi rút ra kết luận giá trị của a 




#410072 $\left\{\begin{matrix}x^2-y+1=0 &...

Đã gửi bởi Tru09 on 03-04-2013 - 10:45 trong Phương trình - hệ phương trình - bất phương trình

Giải hệ phương trình sau:

 

$\left\{\begin{matrix}x^2-y+1=0 & \\ xy+x-2=0 & \end{matrix}\right.$

 

 

Thoạt nhìn, mình tưởng bài này đơn giản rút y theo x thế vào nhưng không được ra nghiệm rất lẻ, còn ý tưởng nhân phương trình trong hệ với số a, b rồi cộng lại thì cũng không được, ra lẻ luôn, ai có ý tưởng nào nữa không vậy?

 

Bài Làm :
$\left\{\begin{matrix}x^2-y+1=0 & \\ xy+x-2=0 & \end{matrix}\right.$ 

Nhân x vào pt đầu rồi cộng với phương trình 2 $\Rightarrow x^3 +2x-2 =0$

Đến đây dùng công thức nghiệm cacdano tính ra x ( hình như có 1 nghiệm là $(\sqrt[3]{1+\sqrt{\frac{35}{27}}} +\sqrt[3]{1-\sqrt{\frac{35}{27}}})$

rồi thay vào tính ra y




#408932 [MSS2013] - Trận 24 - Phương trình nghiệm nguyên

Đã gửi bởi Tru09 on 29-03-2013 - 20:27 trong Thi giải toán Marathon cấp THCS 2013

Giải phương trình nghiệm nguyên

$$x^{2}+ 2x + 4y^{2} = 187$$

 

Đề của 0132345

Bài làm :

$x^2 +2x +4y^2 =187$

$\Leftrightarrow (x+1)^2 +4y^2 =188$

$\Rightarrow$ $4y^2 \leq 188 $

$\Rightarrow y^2 \leq 47$

$\Rightarrow  |y| \leq 6$

Ta thay các giá trị của y :

với $|y| =6 \Rightarrow (x+1)^2 =188 -4.6^2 =44$ (loại )

 

với $|y| =5 \Rightarrow (x+1)^2 =188 -4.5^2 =88$ (loại )

 

với $|y| =4 \Rightarrow (x+1)^2 =188 -4.4^2 =124$ (loại )

 

với $ |y| =3 \Rightarrow (x+1)^2 =188 -4.3^2 =152$ (loại )

 

với $|y| =2 \Rightarrow (x+1)^2 =188 -4.2^2 =172$ (loại )

 

với $|y| =1 \Rightarrow (x+1)^2 =188 -4.1^2 =184$ (loại )

với $|y| =0 \Rightarrow (x+1)^2 =188 -0 =188$ (loại )

Vậy Phương trình đã cho không có nghiệm nguyên .

 
_______________
@Joker: Lời giải đúng
d=10
 
 
 

S = 26+3*10 = 56
 




#406002 [MSS2013] Trận 23 - Hình học

Đã gửi bởi Tru09 on 18-03-2013 - 12:47 trong Thi giải toán Marathon cấp THCS 2013

Lời giải :
A,@~ Ta chứng minh $P,B,Q$ Thẳng hàng .
Gọi giao điểm $MN$ và $OO’$ là I
Gọi $IB \cap (O) =P’$
Ta có :$\Delta IMP’ $~$ \Delta IBM (g-g)$
$\Rightarrow IM^2 =IP’.IB$
Mà xét $\Delta IMO$ có $IM^2 =IH.IO$ (hệ thức lượng )
Vậy $IM^2 = IP’.IB =IH.IO \Rightarrow HOBP’$ là tứ giác nội tiếp .
$\Rightarrow P’ \in (HOB) \Rightarrow (HOB) \cap (O) (1)$
Mà ta có $\angle AOB = 2 \angle O’OB =2\angle APB$
$\Rightarrow \angle O’OB =\angle HPB $
$\Rightarrow HOBP$ là tứ giác nội tiếp .
$\Rightarrow P \in (HOB) \Rightarrow (HOB) \cap (O) (2)$
Từ $(1)$ và $(2) \Rightarrow P \equiv P’ \Rightarrow I,P , B$ thẳng hàng .
Chứng minh tượng tự $\Rightarrow I,B ,Q$ thẳng hàng
Vậy $P,B,Q$ thẳng hàng .
@~ Chứng minh $MP //PN$ và $MB // NQ$
Ta gọi $AB \cap MN = X $
Ta có $XM^2 =XN^2 =XA.XB$
$\Rightarrow XM =XN$
$\Rightarrow X$ là trung điểm MN
Gọi $XA \cap HK =Y$
Xét hình thang $MHKN$ có $XY // NK$ ( $X$ là trung điểm)
$\Rightarrow Y$ là trung điểm $HK$
Mà $AB \perp HK ,YA=YB$
$\Rightarrow AHBK$ là hình thoi
Ta có $\angle HKB = \angle O’QB$
Mà $\angle HKB =\angle KHB$ ( hình thoi )
Mà $\angle KHB =\angle OPB =\angle OBP$
$\Rightarrow \angle OBP =\angle O’QB$
$\Rightarrow OB // O’Q$
Mặt khác $MO // NO’$
Mà $\Delta MOB$ và $\Delta NO’Q$ là 2 tam giác cân
$\Rightarrow \Delta MOB$ ~ $\Delta NO’Q$
$\Rightarrow \angle OMB =\angle O’NQ $
$\Rightarrow MB // NQ ( MO // NO’ (gt))$
Chứng minh tương tự $\Rightarrow MP // NB$
B,
Mà từ $\angle O’QP = \angle OBP ( CMT ) \Rightarrow \angle O’QP = \angle IHP$
$\Rightarrow tứ giác HO’QP$ là tứ giác nội tiếp
$\Rightarrow IH .IO’ =IP .IQ$
Mà tứ giác $MHO’N$ là tứ giác nội tiếp ( có $\angle MHO’ +\angle MNO’ =180^o$)
$\Rightarrow IH .IO’ =IM.IN$
Vậy $ IP .IQ =IM .IN$
Vậy tứ giác $MNQP$ là tứ giác nội tiếp .

### Trường hợp riêng :
Với bán kính $(O) =$ bán kính $(O')$ thì $MN$ không $\cap PQ $.
Khi đó $H \equiv O , K \equiv O' $
$\Rightarrow OO' //PB$
và $OO' // QB$
$\Rightarrow P ,Q,B$ thẳng hàng .
Dễ thấy $MNBP$ và $MNQB$ là 2 hình bình hành
$\Rightarrow MN // NB$ và $MB // NQ$
Và dễ tháy $MNQP$ là hình thang cân $\Rightarrow MNQP$ là tứ giác nội tiếp .

________________________

@Joker: Nộp bài khi hết thời gian

d=0

Hình gửi kèm

  • Lời giải.PNG



#405283 $\begin{Bmatrix} xy^2 -2y +3x^2=0 \\ y^2 +x^2y...

Đã gửi bởi Tru09 on 15-03-2013 - 17:59 trong Phương trình - hệ phương trình - bất phương trình

Giải HPT
$\begin{Bmatrix} xy^2 -2y +3x^2=0 \\ y^2 +x^2y +2x =0 \end{Bmatrix}$



#405248 Chứng minh X,Y,Z thẳng hàng .

Đã gửi bởi Tru09 on 15-03-2013 - 12:09 trong Hình học

Bài toán :
Cho $\Delta ABC$, $M$ là $1$ điểm bất kỳ trong tam giác .Từ $M$ kẻ các đường thẳng vuông góc với $MA,MB,MC$ cắt $BC,AC,AB$ tại $X,Y,Z$
Chứng minh $X,Y,Z$ thẳng hàng :3



#405229 Topic nhận đề Hình học phẳng

Đã gửi bởi Tru09 on 15-03-2013 - 11:16 trong Thi giải toán Marathon cấp THCS 2013

Đề bài :(Tru09)
Cho $(O)$ và $(O')$ cắt nhau ở $2$ điểm $A$ và $B$ . Tiếp tuyến chung $MN$ (gần $A$ hơn ) .Từ $M , N$ kẻ các đường thẳng vuông góc với OO' tại $H$ và $K$ .$AH , AK$ cắt $(O) ,(O')$ tại $P ,Q$ .
a, Chứng minh rằng $MP // NB , MB // NQ$
b, Chứng minh tứ giác $MNQP$ là tứ giác nội tiếp .



#404450 CMR :$0 \leq a+b+c+d-ab-bc-cd-da \leq 2$

Đã gửi bởi Tru09 on 12-03-2013 - 17:01 trong Bất đẳng thức và cực trị

Cho 4 số thực a,b,c,d không âm và nhỏ hơn hoặc bằng 1.
Chứng minh rằng $0 \leq a+b+c+d-ab-bc-cd-da \leq 2$
Khi nào đẳng thức xảy ra ?



#404155 [MSS2013] Trận 22 - PT, HPT đại số

Đã gửi bởi Tru09 on 11-03-2013 - 19:28 trong Thi giải toán Marathon cấp THCS 2013

Chế kiểu gì mà cứ phang bừa cái hằng đẳng thức $a^3 + b^3 + 3ab(a+b)$ với $a,b$ muốn thay cái gì cũng được à :mellow:

Thì có phải bài MSs cũng chính là hằng đẳng thức $a^2 +b^2 =2ab$
Nên tớ mở rộng lên bậc 3 , có gì là không liên quan đến bài MSS !



#404112 [MSS2013] Trận 22 - PT, HPT đại số

Đã gửi bởi Tru09 on 11-03-2013 - 18:09 trong Thi giải toán Marathon cấp THCS 2013

Biết là nói câu này mang tính dìm hàng + ganh ghét nhưng cho hỏi mở rộng này liên quan gì tới bài toàn :wacko:

Mở rộng lên căn bậc 3 , nâng lên hằng đẳng thức (giống như cách giải MSs)



#403917 [MSS2013] Trận 22 - PT, HPT đại số

Đã gửi bởi Tru09 on 10-03-2013 - 23:17 trong Thi giải toán Marathon cấp THCS 2013

Mở rộng 2 :
Giải PT :
$\frac{c}{2} =\sqrt{(a^2-3a+2)x^4 +x^3(ab-2b) +x^2(ac-c-1) +(bc-b)x +c-1} -\frac{(2a-3)x^2 +bx}{2}$
Bài làm :
$PT \Leftrightarrow (2a-3)x^2 +bx +c =2\sqrt{((a-2)x^2 +c-1)((a-1)x^2 +bx +1)} $
Đặt $(a-2)x^2 +c-1 =m$
$(a-1)x^2 +bx +1 =n$
Thay vào ta có :
$m +n =2\sqrt{mn}$
$\Leftrightarrow (\sqrt{m} -\sqrt{n})^2 =0$
$\Leftrightarrow m =n$
$\Leftrightarrow (a-2)x^2 +c-1 = (a-1)x^2 +bx +1$
$\Leftrightarrow -x^2 -bx -2 =0$
$\Leftrightarrow x^2 +bx +2 =0$
Với $-2\sqrt{2} <b<2\sqrt{2}$ thì PT có $\Delta < 0 \Rightarrow$ vô nghiệm
Với b không trong khoảng đó thì
$ x =\frac{-b \pm \sqrt{b^2 -8}}{2}$

 

ĐIểm mở rộng: 6